zl7391e
Thanks Received: 9
Vinny Gambini
Vinny Gambini
 
Posts: 16
Joined: June 03rd, 2011
 
 
 

Q12 - “Though they soon will

by zl7391e Tue Jun 07, 2011 5:59 pm

Hey guys,
The question stem goes "which one of the following, if true, establishes that the doctor's second reason does not cancel out the first?" Now, if the question stem were worded differently, that is, "which one of the following, if true, establishes that the doctor's first reason does not cancel out the second", my question is whether those two are equivalent. If they are, then the question is simply asking "what answer choice can resolve the discrepancy between two reasons offered by the speaker?" Then, why didn't the LSAT makers just put it that way? If test makes have worded the question in 'resolve-the-discrepancy' way, I would have chosen A on this question, for the reason that A) explains both (1) why it's a time-wasting process (because it takes time for doctors to retrieve or even produce patient's records on the spot, as required by the new law) and (2) no patients are going to ask for it (because there is no need to ask since the new law mandates doctors to produce the records when seeing the patients regardless of the patients' preferences).

However, the way that the question stem was worded--the order of the reasons (2nd reason, then 1st reason) was put forward--made me think the correct the answer choice should focus on the second reason. Furthermore, the phrase "if my experience is anything to go by" in the stimulus lure me into thinking the correct answer choice should have something to do with the the speaker's past experience of being a doctor. So I thought the answer choice is something along the line "from my past experience as a doctor, I'm confident that patient's preferences won't get them into asking for access their records in face of the new law. " That is, the new law won't affect their preferences. Therefore, I incorrectly thought the right one should be one from D) or E).

Sorry about the long post about my line of reasoning as how I got the wrong answer choice. I guess my question is how to avoid similar mistake on future practice test? I truly appreciate any input. Also, for those who have done a lot of prep tests, I'm curious about whether the way that question is worded as in this problem (second reason cancel out the first..) common on the lsat? Thanks!
User avatar
 
noah
Thanks Received: 1192
Atticus Finch
Atticus Finch
 
Posts: 1541
Joined: February 11th, 2009
 
This post thanked 7 times.
 
 

Re: Q12 - Though they soon will

by noah Wed Jun 08, 2011 2:43 pm

Good point - those pesky test-writers could have simply asked what resolves the discrepancy.

I don't believe I've seen this sort of question wording in recent exams, so no worries.

In case anyone else is reading this, the discrepancy we're resolving is this: How can it be that the new law giving patients the right to see their records will require lots of work, yet, patients will rarely ask for their records?

(A) resolves that - doctors' teams will have to prepare the records, even if the patient doesn't end up asking for it.

(B) is out of scope - who on the medical team does the work doesn't matter.

(C) simply makes the time burden worse - it doesn't deal with the fact that patients will not ask for their records but the record law will still require a lot of work.

(D) is out of scope - expenses?

(E) is simply boosting one part of the discrepancy - we already learned that patients rarely (if ever) ask for their records.


#officialexplanation